0% found this document useful (0 votes)
9 views32 pages

SolutionsToPracticeProblems

Practice problems for Fina 385

Uploaded by

hotman3473
Copyright
© © All Rights Reserved
We take content rights seriously. If you suspect this is your content, claim it here.
Available Formats
Download as PDF, TXT or read online on Scribd
0% found this document useful (0 votes)
9 views32 pages

SolutionsToPracticeProblems

Practice problems for Fina 385

Uploaded by

hotman3473
Copyright
© © All Rights Reserved
We take content rights seriously. If you suspect this is your content, claim it here.
Available Formats
Download as PDF, TXT or read online on Scribd
You are on page 1/ 32

Solutions to the practice problems

Topic I

Use the following to answer questions 1 and 2:


You have been given this probability distribution for the holding period return for
XYZ stock:
State of the Economy Probability HPR (%)
Boom 0.30 18
Normal growth 0.50 12
Recession 0.20 -5
1. What is the expected holding period return for XYZ stock?
A) 11.67%
B) 8.33%
C) 10.4%
D) 12.4%
E) 7.88%

Solution.
E(rXY Z ) = .3 × 18 + .5 × 12 + .2 × (−5) = 10.4%

2. What is the expected standard deviation for XYZ stock?


A) 2.07%
B) 9.96%
C) 7.04%
D) 1.44%
E) 8.13%

Solution.

σ(rXY Z ) = .3 × (18 − 10.4)2 + .5 × (12 − 10.4)2 + .2 × (−5 − 10.4)2 = 8.13%

3. The standard deviation of return on investment A is 40% while the standard


deviation of return on investment B is 20%. What is the correlation coefficient
between returns on A and B if the covariance of returns on A and B is 0.050?
cov(rA ,rB ) 0.05
Solution. ρ = σA σB
= 0.2×0.4
= 0.625.

4. Consider a portfolio with 70% in stock A and 30% in stock B.


Stock A: E(rA ) = 0.15, σA = 0.20

1
Stock B: E(rB ) = 0.10, σB = 0.5
ρAB = 0.05
Find the expected return and the standard deviation of the portfolio returns

Solution.

E(rP ) = wE(rA ) + (1 − w)E(rB ) = .7 × .15 + .3 × .10 = .135



σP = w2 σA2 + (1 − w)2 σB2 + 2w(1 − w)σA σB ρAB =

.72 × .22 + .32 × .52 + 2 × .7 × .3 × .2 × .5 × .05 = .21

5. In a return-standard deviation space, which of the following statements is (are)


true for risk-averse investors? (The vertical and horizontal lines are referred to as the
expected return-axis and the standard deviation-axis, respectively.)
I) An investor’s own indifference curves might intersect.
II) Indifference curves have negative slopes.
III) In a set of indifference curves, the highest offers the greatest utility.
IV) Indifference curves of two investors might intersect.

A) I and II only
B) II and III only
C) I and IV only
D) III and IV only
E) none of the above

Solution.
D), since an investor’s own indifference curves can not intersect and they are
upward sloping

6. Consider portfolio A that yields 12% return and has a standard deviation of 40%
and portfolio B that yields 15% return and has a standard deviation of 50%. Which
portfolio is better for an investor whose coefficient of risk aversion is 4? Suppose
that investor Abdul is indifferent between portfolios A and B. What should be his
coefficient of risk aversion?

Solution.
Utility function from portfolio A is
1
UA = 0.12 − 4 × 0.42 = −0.2
2

2
Utility function from portfolio B is
1
UA = 0.15 − 4 × 0.52 = −0.35
2
It follows that portfolio A is better for the investor with A = 4.
Abdul is indifferent between portfolios A and B if UA = UB , or
1 1
0.12 − A × 0.42 = 0.15 − A × 0.52
2 2
Solution of the last equation with respect to A is A(0.25 − 0.16) = 0.30 − 0.24 →
A = 0.06/0.09 = 0.67.

7. Suppose you currently hold a portfolio that yields 15% return and has a
standard deviation of 20%. What should the minimum rate of return on a portfolio
with standard deviation of 25% be to make you indifferent between the two portfolios?
Assume your coefficient of risk aversion is 3.

Solution.
Investor is indifferent between the two portfolios if they provide the same utility.
Utility from the first portfolio:
U1 = .15 − 12 × 3 × (.2)2 = .09
Utility from the second portfolio:
U2 = E(r2 ) − 12 × 3 × (.25)2 = E(r2 ) − .09375
Therefore, an investor is indifferent between the two portfolios if E(r2 ) − .09375 =
.09, or E(r2 ) = .18375

8. A portfolio is comprised of two stocks, A and B. Stock A has a standard


deviation of return of 20% while stock B has a standard deviation of return of 5%.
Stock A comprises 70% of the portfolio while stock B comprises 30% of the portfolio. If
the variance of the return on the portfolio is 0.0230, what is the correlation coefficient
between returns on A and B?

Solution.
The variance of the whole portfolio is

σP2 = w2 σA2 + (1 − w)2 σB2 + 2w(1 − w)σA σB ρAB

Therefore,

σP2 − w2 σA2 − (1 − w)2 σB2 .023 − .72 × .22 − .32 × .052 .003175
ρAB = = = = 0.76
2w(1 − w)σA σB 2 × .7 × .3 × .2 × .05 .0042

3
Topic II
1. An investor invests 30% of his wealth in a risky asset with an expected rate of
return of 0.15 and a variance of 0.04 and 70% in a T-bill that pays 6%. His portfolio’s
expected return and standard deviation are
A) 0.114 and 0.12
B) 0.087 and 0.06
C) 0.295 and 0.12
D) 0.087 and 0.12
E) none of the above,
respectively

Solution.

E(rC ) = yE(rP ) + (1 − y)rf = .3 × .15 + .7 × .06 = .087


σC = yσP = .3 × .2 = 0.06

2. You invest $1000 in a complete portfolio which is comprised of a risky asset


with an expected rate of return of 12% and a standard deviation of 20% and a T-bill
with a rate of return of 5%. If you want your complete portfolio to have a standard
deviation of 10%, how much should you invest in T-bills? What would the expected
return on that portfolio be?

Solution.
From σC = yσP = y × 0.2 = .1 we find y = 0.5 and 1 − y = .5

E(rC ) = yE(rP ) + (1 − y)rf = .5 × .12 + .5 × .05 = .085

3. You have $1, 000, 000 to invest. The risk-free rate as well as your borrowing
rate is 2%. The return on the risky portfolio is 12%. If you wish to earn a 22% return,
how much should you borrow?

Solution.
E(rC ) = yE(rP ) + (1 − y)rf = y × .12 + (1 − y) × .02 = .02 + y × .1 = .22 implies
y = 2.
Therefore, investor has to borrow $1, 000, 000.

4. Consider a portfolio that has two risky securities A and B. Annual standard
deviations of these securities are σA = 20% , σB = 40%, respectively, while the

4
correlation coefficient between them is −1. Find the wights of securities A and B in
the portfolio if this portfolio is risk–free.

Solution. Standard deviation of the portfolio is

σP = wA σA − (1 − wA )σB

Because σP = 0, we arrive to equation for wA :


σB
0 = wA σA − (1 − wA )σB → wA =
σA + σB
and wB = 1 − wA = σA
σA +σB
.

5. You are considering investing $1, 000 in a complete portfolio. The complete
portfolio is comprised of T-bills that pay 3% and a risky portfolio P, constructed
from 2 risky securities X and Y. The weights of X and Y in P are 80% and 20%,
respectively. X has expected rate of return of 15% and Y has an expected rate of
return of 8%. To form a complete portfolio with an expected rate of return of 10%,
how much should you invest in T -bills, security X and security Y?

Solution.
Expected return of the risky portfolio

E(rP ) = wX E(rX ) + wY E(rY ) = .8 × 15 + .2 × 8 = 13.6

Return of the complete portfolio

E(rC ) = rf + y[E(rP ) − rf ] = 3 + y × 10.6 = 10

Thus, y = .66, 1 − y = .34. Fractions of the complete portfolio invested in stock X


and Y are .66 × .8 = .528 and .66 × .2 = .132, respectively.

6. An investor can design a risky portfolio based on two stocks, K and L. Stock
K has an expected rate of return of 18% and a standard deviation of return of 30%.
Stock L has an expected rate of return of 14% and a standard deviation of return of
5%. The correlation coefficient between the two stocks is 0.5. The risk-free rate is
5%.
(a) what is the weight of stock K in the optimal risky portfolio?
(b) what is the expected return on the optimal risky portfolio?
(c) what is the standard deviation of return on the optimal risky portfolio?

Solution.

5
(a) We know
⋆ SK − ρSL
wK =
SK + SL σL − ρ(SL +
σK
SK σσKL )
Substituting our data into the last equation gives

SK = (0.18 − 0.05)/.30 = 0.433


SL = (0.14 − 0.05)/.05 = 1.8

wK = −0.052
wL⋆ = 1 − (−.052) = 1.052
E(rP ) = (−0.052) × 0.18 + 1.052 × 0.14 = 0.1379

σP = .0522 × 0.32 + 1.0522 × .052 + 2 × (−0.052) × 1.052 × .3 × .05 × 0.5 = 0.0468

7. When borrowing and lending at a risk-free rate are allowed, which Capital
Allocation Line (CAL) should the investor choose to combine with the efficient
frontier?
I) with the highest reward-to-variability ratio.
II) that will maximize his utility
III) with the steepest slope.
IV) with the lowest slope.
A) I and III
B) I and IV
C) II and IV
D) I only
E) I, II, and III

Solution.
E) The slope of CAL is the reward-to-variability ratio and it should be the highest.
The highest slope also maximizes the expected utility of an investor.

8. Consider an optimal portfolio with two risky assets such that each asset has
positive Sharpe ratio. Is it possible that an investor will short-sell one of these two
assets in this portfolio to take a leveraged position in the other? Why?
A) No it is not possible because y* is positive if the Sharpe ratio of an asset is
positive
B) No it is not possible because an investor cannot short an asset in the market
C) No it is not possible because an investor will not short an asset which price is
likely to increase in the future

6
D) Yes it is possible because the price of the shorted asset is likely to fall in the
future
E) Yes it is possible if correlation between two assets is significant since short-
selling will improve the portfolio return-to-risk characteristics

Solution. See practice problem on page 45. It follows from this practice problem
that investor will short security even if its Sharpe ratio is positive given the presence
of another security with a higher Sharpe ratio. Answer is E.

9. Suppose that you have two stocks K and L that have correlation coefficient
more than −1 and less than 1. Assume that the portfolio opportunity set when short–
selling is allowed is known. Now let us assume that stocks cannot be sold short. What
would the portfolio opportunity set look like in this case?
What would be the portfolio opportunity set if you could sell short stock K, but
not stock L?

L
E(r)

Solution. Draw portfolio opportunity set through points K and L. Let portfolio
A be on this set and lie above security L and on its right side. This portfolio has
a short allocation to K and a long allocation to L (see question on page 35 of the
notes). Therefore, this portfolio is not feasible since short positions are not allowed.
Now consider portfolio B that lies on the portfolio opportunity set that lies below
K. This portfolio has a short position in L and a long position in K. Therefore, it is
not allowed either.
Finally, consider portfolio C on the portfolio opportunity set that lies between K
and L. It has long positions in both securities K and L. Therefore, it is feasible.

7
We conclude that a new portfolio opportunity set is a part of the original portfolio
opportunity set located between securities K and L

10. The following information applies to questions a to d:


Consider a market with three mutual funds. The first is a stock fund, the second
is a long–term corporate bond fund, and the third is a T–bill money market fund
that yields a rate of 0.5 percent. All investors have identical beliefs and the expected
returns and standard deviations of returns are:

Expected Return Standard Deviation


Stock fund (S) .165 .50
Bond fund (B) .08 .25

The correlation between the fund returns is .4.

a. Solve numerically for the proportions of each asset in the optimal risky portfolio
P (portfolio that includes only risky assets). Find the expected return and standard
deviation of this portfolio.

b. You require that your complete portfolio (portfolio that includes T-bills and
risky funds) yield an expected return of 19 percent and be efficient on the best feasible
CAL. What is the proportion invested in the T-bill fund and each of the two risky
funds? What is the standard deviation of your complete portfolio?

c. If you were to use only the two risky funds (no T-bills) and will require an
expected return of 19 percent, what must be the investment proportions of your
portfolio? Compare its standard deviation to that of the optimized portfolio in
problem 2. What do you conclude?

d. Now assume that Michael has coefficient of risk aversion equal to 1. Find the
weight of each security in his complete portfolio (portfolio that includes T-bills and
risky portfolio P).

Solution:
a. By using the formulas for the weights of risky securities in portfolio P with Sharpe
ratios we find:
wS = .3676, wB = .6324

8
E(rP ) = 11.12%, σP = 28.64%

b. By using the formula for the expected return of complete portfolio with T-bills
and portfolio P we write:

19 = 0.5 + y(11.12 − 0.5).

Hence, y=1.742. It follows that

yS = 1.742 ∗ 0.3676 = .640, yB = 1.742 ∗ 0.6324 = 1.102, yT −bill = −.742

σC = 1.742 ∗ 28.64 = 49.89%

c. By using the formula for the expected return of portfolio with two risky
securities we write:
19 = 8 + wS (16.5 − 8).
Hence,
wS = 1.294, wB = −.294
and
σP = 62.13%

d. The wight of portfolio P in the complete portfolio is given by the formula for

y . Hence:
0.1112 − 0.005
y∗ = = 1.295
1 ∗ 0.28642
It follows that

yS = y ∗ ∗ wS = .476, yB = y ∗ ∗ wB = .819, yT −bill = −.295

11. An investor with a coefficient of risk aversion of 3 can design a risky portfolio
based on two stocks, K and L. Stock K has an expected rate of return of 25% and a

9
standard deviation of return of 30%. Stock L has an expected rate of return of 10%
and a standard deviation of return of 12%. The correlation coefficient between the
two stocks is 0.4. The risk free lending rate is 4% while the risk free borrowing rate
is 7%.
a. What is the weight of stock K in the optimal risky portfolio if an investor
lends? What are the expected return and the standard deviation of return on this
portfolio?
b. What is the weight of stock K in the optimal risky portfolio if an investor
borrows? What are the expected return and the standard deviation of return on this
portfolio?
c. Find weights of all assets in the optimal complete portfolio for the investor
whose coefficient of risk aversion is 3.

Solution:

a. By using the formulas for the weights of risky securities in portfolio P with
Sharpe ratios and a lending rate we find:
SK − ρSL 0.7 − 0.4 × 0.5
wK = = = 0.476,
SK + σK
S − ρ(SL + σσKL SK )
σL L 0.7 + 30
12
0.5 − 0.4(0.5 + 30
12
0.7)

where SK = 25−4
30
= 0.7, SL = 10−4
12
= 0.5. The weight allow us to find the expected
return and the standard deviation of returns of portfolio P:

E(rP ) = 17.14%, σP = 17.756%

b. By using the formulas for the weights of risky securities in portfolio P with
Sharpe ratios and a borrowing rate we find:
SK − ρSL 0.6 − 0.4 × 0.25
wK = = = 0.952,
SK + σK
S − ρ(SL + σσKL SK )
σL L 0.6 + 30
12
0.25 − 0.4(0.25 + 30
12
0.6)

where SK = 25−7
30
= 0.6, SL = 10−7
12
= 0.25. The weight allow us to find the expected
return and the standard deviation of returns of portfolio P:

E(rP ) = 24.28%, σP = 28.795%

c. Assume that investor lends then the weight of portfolio P in her/his complete
portfolio is
0.1714 − 0.04
y∗ = = 1.389
3 ∗ 0.177562

10
We arrive at contradiction since y ∗ should be less than 1 if investor lends. Assume
that investor borrows then the weight of portfolio P in her/his complete portfolio is
0.2428 − 0.07
y∗ = = 0.695
3 ∗ 0.287952
We arrive at contradiction again since y ∗ should be more than 1 if investor borrows.
We concluder that the investor neither borrows nor lends. It follows that the investor
buys a portfolio which lies on the portfolio opportunity set. The portfolio includes
only positions in security 1 and 2. By using the formulas for the weights of risky
securities in portfolio P in the market with no T-bills we find:

yK = .6614, yL = .3386, yT −bill = 0

12. Consider portfolio P that is comprised from two stocks, A and B. Stock A has
a standard deviation of return (σA ) of 15% while stock B has a standard deviation of
return (σB ) of 40%. The correlation coefficient between the two stocks (ρ) is 0.6.
a. Graph the relation between the weight of stock A and the standard deviation
of portfolio P. Show the weight of stock A along the horizontal axis.
b. Find the weights of stock A and B in the minimal variance portfolio.

Solution:

a. See the class notes (Topic 2 Portfolio with two risky assets)

b. We use the formula for the weight of stock A in the minimum variance portfolio:

⋆ σB2 − Cov(rB , rA )
wA = .
σA2 + σB2 − 2Cov(rB , rA )
We find after plugging in the numbers that

wA = 1.12

Topic III
1. Which statement is not true regarding the market portfolio?
A) It includes all publicly traded financial assets.

11
B) It lies on the efficient frontier.
C) All securities in the market portfolio are held in proportion to their market
values.
D) It is the tangency point between the capital market line and the indifference
curve.
E) all of the above are true.

Solution.
D) The tangency point between the capital market line and the indifference curve
is a complete portfolio of an investor which includes a risk–free asset.

2. Your personal opinion is that security X has an expected rate of return of 0.11.
It has a beta of 1.5. The risk-free rate is 0.05 and the market expected rate of return
is 0.09. According to the Capital Asset Pricing Model, this security is
A) underpriced.
B) overpriced.
C) fairly priced.
D) cannot be determined from data provided.
E) none of the above.

Solution. C) α = E(ri )−[rf +βi [E(rM )−rf ]] = 0.11−[0.05+1.5(0.09−0.05)] = 0

3. According to CAPM, the fair return of the stock is 15%.


(a) What is the stock’s beta if the risk–free rate is 3% and the market return is
12%?
(b) What is the standard deviation of the return on the market portfolio if
cov(ri , rM ) = 0.06

Solution.
(a) From E(ri ) = rf + βi [E(rM ) − rf ] we find

E(ri ) − rf 15 − 3
βi = = = 1.33
E(rM ) − rf 12 − 3

cov(ri ,rM )
(b) From βi = σM2 we find

2 cov(ri , rM ) 0.06
σM = = = 0.045
βi 1.33

and σM = 15.97%

12
4. The fair expected return of the stock is 15%. Find the market risk premium
if the risk–free rate is 5% and the stock’s beta is 0.9. What is the market expected
return?

Solution.
From E(ri ) = rf + βi [E(rM ) − rf ] we find

E(ri ) − rf 15 − 5
E(rM ) − rf = = = 11.1%,
βi 0.9
E(rM ) = 11.1 + 5 = 16.1%

5. Consider the following table, which gives historical returns on two stocks and
the market: Assume that these returns can be used to estimate expectation, variance

Year Market return Aggressive stock Defensive stock


2011 0.09 0.05 0.08
2012 0.12 0.13 0.12
2013 0.18 0.24 0.16

and covariance of future returns.


a. What are the expected rates of return on each stock and the market?
b. What are the betas of the two stocks?
c. If the T-bill rate is 1 percent, draw the SML for this economy.
d. Plot the two securities on the SML graph. What are the alpha’s of each stock?

Solution. a. We find estimates of the expected returns by using the historical


means:
1
r̄M = (9 + 12 + 18) = 13%,
3
1
r̄A = (5 + 13 + 24) = 14%,
3
1
r̄D = (8 + 12 + 16) = 12%
3
Cov(ri ,rM )
b. Since βi = 2
σM
, we have to find the estimates of variance and covariance:

1
Cov(rA , rM ) = [(14 − 5)(13 − 9) + (14 − 13)(13 − 12) + (14 − 24)(13 − 18)] = 43.5(%)2
2
1
Cov(rD , rM ) = [(12 − 8)(13 − 9) + (12 − 12)(13 − 12) + (12 − 16)(13 − 18)] = 18(%)2
2
1
2
σM = [(13 − 9)2 + (13 − 12)2 + (13 − 18)2 ] = 21(%)2
2
13
It follows that
43.5
βA = = 2.071,
21
18
βD = = 0.857
21
c. Draw a straight line through points (0, 1) and (1, 13) on E(r) − β plane
d.
αA = 14 − (1 + 2.071(13 − 1)) = −11.852%
αD = 12 − (1 + 0.857(13 − 1)) = 0.716%

6. Use the following information on the returns of market, rM , and returns of


stock j, rj :

State Probability rM rj
1 0.1 -0.15 -0.30
2 0.3 0.05 0.00
3 0.35 0.15 0.20
4 0.25 0.2 0.50

Find alpha of stock j if rf = 3%.

Solution. According to CAPM, E(rj ) = rf + βj (E(rM ) − rf ). It follows from the


table that

E(rM ) = −0.15 ∗ 0.1 + 0.05 ∗ 0.3 + 0.15 ∗ 0.35 + 0.2 ∗ 0.25 = 0.1025

E(rj ) = −0.30 ∗ 0.1 + 0.0 ∗ 0.03 + 0.20 ∗ 0.35 + 0.5 ∗ 0.25 = 0.165
2
σM = 0.1∗(−15−10.25)2 +0.3∗(5−10.25)2 +0.35∗(15−10.25)2 +0.25∗(20−10.25)2 = 103.6875(%)2
Cov(rj , rM ) = 0.1 ∗ (−15 − 10.25)(−30 − 16.5) + 0.3 ∗ (5 − 10.25)(0 − 16.5)+
0.35 ∗ (15 − 10.25)(20 − 16.5) + 0.25 ∗ (20 − 10.25)(50 − 16.5) = 230.875(%)2
Therefore,
Cov(rj , rM ) 230.875
βj = 2
= = 2.227
σM 103.6875
and
E(rj ) = 19.14%

14
It follows that
αj = −2.64%

Topic IV
1. Suppose you held a well-diversified portfolio with a very large number of
securities, and that the single index model holds. If the σ of your portfolio was 0.20
and σM was 0.16, the β of the portfolio would be approximately
A) 0.64
B) 0.80
C) 1.25
D) 1.56
E) none of the above

Solution. C): Because σ 2 ≈ β 2 σM


2
, we find β = σ/σM = .2/.16 = 1.25

2. Consider the single factor model. Portfolio A has a beta of 0.2 and an expected
return of 13%. Portfolio B has a beta of 0.4 and an expected return of 15%. The
risk-free rat of return is 10%. If you wanted to take advantage of an arbitrage
opportunity, you should take a short position in portfolio and a long position
in portfolio
A) A, A
B) A, B
C) B, A
D) B, B
E) none of the above

Solution. C): Portfolio A: rA = 13% + 0.2F ; Portfolio B: rB = 15% + 0.4F . Make


portfolio C from Portfolio B and a risk free asset with equal weights. Then βC = 0.2
and rC = 0.5rB + 0.5rf = 7.5% + 0.2F + 5% = 12.5% + 0.2F . Because portfolios A
and C have the same risk but A has higher expected return, we short C (and so B)
and take a long position in A.

3. Assume that security returns are generated by the single–index model

Ri = αi + βi RM + ei ,

15
where Ri is the excess return for security i, and RM is the market’s excess return. The
risk–free rate is 3%. Suppose that there are three securities, A and B characterized
by the table below

Security βi E(Ri ), % σ(ei ), %


Stock A 1.25 12.5 25
Stock B 1.5 12 35

a. If σM = .20, calculate the variance of returns of securities, A and B.


b. Now assume that there is an infinite number of assets with return characteristics
identical to those of A and B, respectively. If one forms a well-diversified portfolio of
type A securities, what will be the mean and variance of the portfolio excess returns?
What about portfolios composed only of type B stocks?
c. Are there arbitrage opportunities in the market of question b? If so, what
would be an arbitrage strategy?
d. Now consider a single security A from question a and a well diversified portfolio
B from question b. Are there arbitrage opportunities in this market? If so, what
would be an arbitrage strategy?

Solution:
a. Use formula for variance of a security which return follows a single–index model:
σi2 = βi2 σM
2
+ σ(ei )2 to find:

σA2 = 1250%2 , σA2 = 2125%2

b. For a well-diversified portfolio which follows a single–index model the variance is


given by σi2 = βi2 σM
2
. Hence,

σA2 = 625%2 , σB2 = 900%2

E(RA ) = 12.5%, E(RA ) = 12%

c. Slope of security A is 10, slope of security B is 8. Arbitrage strategy: short portfolio


B and buy portfolio C in which wA = 1.2 and wT −bill = −.2.

d. There are no arbitrage strategies since security–specific risk of A will affect any
strategy which includes security A, portfolio B and T-bills.

16
4. Assume that the single index model holds for all securities and an investor
comes up with the following equation for the return of the well diversified portfolio
P:
rP = 17% + 0.8Rm ,
where Rm is an excess return on the market. The risk free rate is 2% and the market
expected return is 10%.
a. Does APT hold for portfolio P?
b. Does an arbitrage opportunity exist in this economy? If so, what would be an
arbitrage strategy?
c. Now suppose that portfolio P is not well–diversified so that

rP = 17% + 0.8Rm + eP ,

where eP is unexpected contribution from portfolio specific risk to the return of P .


Does an arbitrage opportunity exist? Why? If yes, what is an arbitrage strategy?
Solution. a. Let us find E(rP ). We find it by taking expectation from the left
and the right side of the equation for r:

E(rP ) = 17% + 0.8(E(rm ) − rf ) = 17 + 0.8(10 − 2) = 23.4%

Yet, the APT predicts: E(rP ) = rf + β(E(rm ) − rf ) = 2 + 0.8(10 − 2) = 8.4% It


follows that APT does not hold, that is α ̸= 0.

b. Arbitrage exists. Let the market portfolio be a benchmark. We construct


portfolio C from T-bills and P:

βC = βM = 1 = wP βP

implying that
1 1
wP = = = 1.25
βP 0.8
and wT −bill = −0.25. It follows that

E(rC ) = 1.25 × 23.4 + (−0.25)2 = 28.75%

which is bigger than the expected return on the market implying that the market
portfolio is overpriced with respect to portfolio C. It follows that investor should
short the market portfolio and buy portfolio C.

17
c. Arbitrage opportunity does not exist since portfolio p is not well–diversified.
Any combination of this portfolio with the market portfolio and T-bills entitles
investor with a portfolio–specific risk and, therefore, a possibility of loosing money.

5. Consider the multifactor APT with two factors. Stock A has an expected
return of 16.4%, a beta of 1.4 on factor 1 and a beta of .8 on factor 2. The risk
premium on the factor 1 portfolio is 3%. The risk-free rate of return is 6%. What is
the risk-premium on factor if no arbitrage opportunities exit?

Solution. D) :
16.4% = 1.4(3%) + .8x + 6% → x = 7.75.

Use the following to answer questions 6 and 7:


Consider the multifactor APT. There are two independent economic factors, F1
and F2 . The risk-free rate of return is 6%. The following information is available
about two well-diversified portfolios:
Portfolio β on F1 β on F2 Expected Return (%)
A 1.0 2.0 19
B 2.0 0.0 12

6. Assuming no arbitrage opportunities exist, the risk premium on the factor F1


portfolio should be
A) 3%
B) 4%
C) 5%
D) 6%
E) none of the above

Solution. Use the APT equation for portfolio B to find the risk premium on
factor portfolio 1: 12% = 6% + 2.0(RP1 ) + 0(RP2 ) → RP1 = 3%. Now use
the APT equation for portfolio A to find the risk premium on factor portfolio 2:
19% = 6% + 1.0 × 3.0% + 2.0(RP2 ) → RP1 = 5%

7. Assuming no arbitrage opportunities exist, the risk premium on the factor F2


portfolio should be
A) 3%
B) 4%
C) 5%

18
D) 6%
E) none of the above

Solution. C): See the previous solution

8. Random returns for two well–diversified portfolios at time t are given by:

rAt = 0.27 + 2F1t + 0.8F2t


rBt = 0.161 + F1t + 1.1F2t ,
where F1 and F2 are unexpected parts of factor 1 and 2 returns, respectively. (One
can think that factor one is GDP and factor two is an inflation). The risk free rate is
1.0%.
a. Construct factor portfolio for factor 1 by combining portfolios A, B, and T-bills.
What are the weights of these portfolios in factor portfolio 1? What is the expected
return of factor portfolio 1?
b. Solve question a for factor portfolio 2
c. Assume that the market does not allow arbitrage strategies and so the two–
factor APT holds. Find the expected returns on portfolio C which betas with respect
to factors 1 and 2 are 0.5 and 1.2, respectively.

Solution:
a. Factor portfolio 1 must have β1 = 1, β2 = 0. One can write that

β1 = wA βA1 + wB βB1

β2 = wA βA2 + wB βB2
which give us the system with 2 equations:

1 = 2wA + wB

0 = 0.8wA + 1.1wB
The solution of this system is:

wA = .785, wB = −.571.

It follows that

yT −bill = .785, E(r1 ) = wA E(rA ) + wB E(rB ) + WT −bill rf = 12.79%

19
b. Factor portfolio 2 must have β1 = 0, β2 = 1, which give us the system with 2
equations:
0 = 2wA + wB
1 = 0.8wA + 1.1wB
The solution of this system is:

wA = −.714, wB = 1.428.

It follows that
yT −bill = .286, E(r1 ) = 4.00%

c. Use APT equation: E(rC ) = rf + βC1 (E(r1 ) − rf ) + βC2 (E(r2 ) − rf ) to get

E(rC ) = 10.49%

Topic V
1. A coupon bond that pays interest of $100 annually has a par value of $1, 000,
matures in 5 years, and is selling today at a $72 discount from par value. The yield
to maturity on this bond is
A) 6.00 %
B) 8.33 %
C) 12.00 %
D) 60.00 %
E) none of the above
100 100 100 1000
Solution. C. Check: 1.12
+ 1.122
+ ... + 1.125
+ 1.125
= $927.9

2. You purchased an annual interest coupon bond one year ago that had 6 years
remaining to maturity at that time. The coupon interest rate was 10% and the par
value was $1, 000. At the time you purchased the bond, the yield to maturity was
8%. If you sold the bond after receiving the first interest payment and the yield to
maturity continued to be 8%, your annual total rate of return on holding the bond
for that year would have been
A) 7.00 %
B) 7.82 %
C) 8.00 %
D) 11.95 %

20
E) none of the above

Solution. C: Price of the bond one year from now is


100 100 100 1000
P1 = + 2
+ ... + 5
+ = $1079.85
1.08 1.08 1.08 1.085
Price of the bond today is
100 100 100 100 1000
P1 = + 2
+ ... + 5
+ 6
+ = $1092.46
1.08 1.08 1.08 1.08 1.086
1,079.85−1,092.46+100
HP R = 1,092.46
= 8%

3. Bond of Zello Corporation pays its coupons semiannually. The annual coupon
rate is 6%, the maturity is 15 years, and the annual yield to maturity is 7%.
i. Find the holding period return for one year investment period if the bond is selling
at annual yield to maturity 6% by the end of the year and semiannual reinvestment
rate is 3%.
ii. Find the realized compound yield for a two–year holding period, assuming that
(1) you sell the bond after two years; (2) the bond annual yield is 6.5% at the end of
the second year; and (3) semiannual reinvestment rate remains at 3.5% for the next
two years.

Solution.
i.
P1 + C(1 + r) + C
HP R = − 1 = 16.83%,
P0
where r is the reinvestment rate, P0 = $908.04, and P1 = 1, 000.00. Price P0 is found
with the help of financial calculator and price P1 is found from the condition that the
coupon rate and the yield to maturity become equal in 1 year.
ii. [( )1/4 ]
FV
RCYa = 2 − 1 = 9.01%,
P0
where
F V = P2 + C(1 + r)3 + C(1 + r)2 + C(1 + r) + C,
r is semiannual reinvestment rate and

P2 = $956.57, F V = 1, 083.02

Price P2 is found with the help of financial calculator after taking into account that
time to maturity of the bond is 13 years

21
4. Consider a bond that makes n semiannual coupon payments with semiannual
coupon rate c. The time before the next coupon payment is t which is less than half
a year. The semiannual YTM of the bond is y which is equal to c. Find the price of
the bond in terms of its par value. Is the bond at par, at premium, or at discount?
Show all your steps for the full credit.

Solution.
[ ]
1 − (1 + y)−n
P = P arV × (1 + y)1−t/τ
cr + (1 + y)−n
y

= P arV × (1 + y)1−t/τ > P arV

Use the following to answer questions 5-7:


The following is a list of prices for zero coupon bonds with different maturities
and par value of $1, 000.
Maturity (Years) Price ($)
1 943.40
2 881.68
3 808.88
4 742.09

5. What is, according to the expectations theory, the expected forward rate in
the third year?
A) 7.00 %
B) 7.33 %
C) 9.00 %
D) 11.19 %
E) none of the above

Solution. C: 881.68/808.88 − 1 = 9%

6. What is the yield to maturity on a 3-year zero coupon bond?


A) 6.37 %
B) 9.00 %
C) 7.33 %
D) 10.00 %
E) none of the above

22
Solution. C: (1000/808.81)1/3 − 1 = 7.33%

7. What is the price of a 4-year maturity bond with a 12% coupon rate paid
annually? (Par value = $1, 000)
A) $742.09
B) $1, 222.09
C) $1, 000.00
D) $1, 141.84
E) none of the above

Solution. Interest rates: y1 = (1000/943.40)−1 = 6.00%, y2 = (1000/881.68)1/2 −


1 = 6.50%, y3 = (1000/808.88)1/3 − 1 = 7.33%, y4 = (1000/742.09)1/4 − 1 = 7.742%
120 120 120 1120
Price P = 1.06 + 1.065 2 + 1.07333 + 1.077424 = $1147.21

8. The yield to maturity on one-year zero-coupon bonds is currently 5 percent;


the annual YTM on two-year zero–coupon bond is 6 percent. The federal government
plans to issue a two-year maturity coupon bond, paying coupons once per year with
an annual coupon rate of 8 percent. The face value of the bond is $1000.
a. At what price will the bond sell?
b. What will be the yield to maturity on the bond?
c. If the expectations theory is correct, what is the market expectation of the
price that the bond will sell for next year?
d. Recalculate your answer to (c) if you believe in the liquidity preference theory
and you believe that the liquidity premium is 1 percent.

Solution.

80 1080
a. P = + = $1037.39
1.05 (1.06)2
b. y = 5.96%
1080 1080
c. P = = = $1009.26,
1 + r2 1 + f2
(1.06)2
where f2 = 1.05
− 1 = 7.00%

1080 1080
d. P = = = $1018.77,
1 + r2 1 + f2 − ∆
where ∆ is the liquidity premium

23
9. Consider the bond market where the current semiannual spot rates for 6
months, 12 months and 18 months are y1 = 2.1%, y2 = 2.2%, and y3 = 2.3%,
respectively. Find annual realized compound yield for 1.5 years for semiannual coupon
bond with annual coupon rate 5% and face value $1000 if:
a) an investor uses expectation hypothesis,
b) an investor uses liquidity preference hypothesis in which the semiannual liquidity
premium is equal to 0.5% for any future time interval.

Solution. We need to find the price of the bond today and reinvestment rates
to find RCY. The reinvestment rates are equal to the short interest rates for the
corresponding time intervals in the future. We find these rates by using expectation
or liquidity-preference hypothesis.
The fair price of the bond today is
25 25 1025
P = + 2
+ = $1005.83
1 + y1 (1 + y2 ) (1 + y3 )3

a) In the case of the expectation hypothesis, rn = fn , hence we have to find the


forward rates for the following time intervals: [0.5, 1.5] and [1, 1.5]. The semiannual
forward rate for time interval [0.5, 1.5] is

f1,3 = (1 + f2 )(1 + f3 ) − 1

and the semiannual forward rate for time interval [1, 1.5] is f3 , where

(1 + y2 )2 (1 + 0.022)2
1 + f2 = = = 1.023
1 + y1 1 + 0.021

(1 + y3 )3 (1 + 0.023)3
1 + f3 = = = 1.025.
(1 + y1 )2 (1 + 0.022)2
It follows that the short interest rates are
√ √
r1,3 = f1,3 = (1 + f2 )(1 + f3 ) − 1 = 1.023 · 1.025 − 1 = 2.40%,

r3 = f3 = 2.50%.
Hence, the future value of the cash flow for the bond is

F V = 25(1 + r1,3 )2 + 25(1 + r3 ) + 1025 = $1, 076.84

and semiannual RCY is


[ ]1/3
1, 076.84
RCY = − 1 = 2.30%
1, 005.83

24
b) In the case of the liquidity-preference hypothesis, rn = fn − ∆n , where
∆n = 0.5%. Hence, we find
√ √
r1,3 = f1,3 − ∆1,3 = (1 + f2 )(1 + f3 ) − 1 − 0.005 = 1.023 · 1.025 − 1.005 = 1.90%,

r3 = f3 − 0.005 = 2.00%.
Hence, the future value of the cash flow for the bond is

F V = 25(1 + r1,3 )2 + 25(1 + r3 ) + 1025 = $1, 076.46

and semiannual RCY is


[ ]1/3
1, 076.46
RCY = − 1 = 2.29%
1, 005.83

The difference between RCY’s in questions a and b is very small due to a short
time to maturity of the bond.

10. Consider the bond market with the following two semiannual coupon-paying
bonds

Maturity, years annual coupon rate, % Price, $


0.5 6 1015
1 5 1030
Find 6 months and 12 months spot rates in the market.

Solution. Let us write the formulas for the prices of the two bonds:
1030
1015 = ,
1 + y6
25 1025
1030 = + ,
1 + y6 (1 + y12 )2

where the rates y6 and y12 are semiannual. The solution of the first equation reveals
1030
y6 = − 1 = 1.48%.
1015
Then the solution of the second equation reveals

1025
y12 = − 1 = 0.97%.
1030 − 1+0.0148
25

25
11. Consider two annual coupon bonds, each with two years to maturity. Bond
A has a 7% coupon and a price of $1, 000.62. Bond B has a 10% coupon and sells for
$1, 055.12. Find the two one-period forward rates that must hold for these bonds.
A) 6.97%, 6.95%
B) 6.95%, 6.95%
C) 6.97%, 6.97%
D) 6.08%, 7.92%
E) 7.00%, 10.00%

Solution. D. One has to solve

1, 000.62 = d1 × 70 + d2 × 1, 070

1, 055.12 = d1 × 100 + d2 × 1, 100


where d1 = 1/(1 + r1 ), d2 = 1/(1 + r2 )2
We find d2 = .8735; d1 = .9427 which implies r1 = 6.08%; r2 = 7.00%. Hence the
forward rates are r1 = 6.08% and f2 = (1.07)2 /1.06 = 7.92%

12. Holding other factors constant, which one of the following bonds has the
smallest price volatility?
A) 5-year, 0% coupon bond
B) 5-year, 12% coupon bond
C) 5 year, 14% coupon bond
D) 5-year, 10% coupon bond
E) Cannot tell from the information given.

Solution. C: Duration (and thus price volatility) is lower when the coupon rates
are higher.

13. The duration of a par value bond with a coupon rate of 8% and a remaining
time to maturity of 5 years is
A) 5 years.
B) 5.4 years.
C) 4.17 years.
D) 4.31 years.
E) none of the above.

Solution. D
Calculations are shown below:

26
Yr. CF, $ PV of CF at 8%, $ Weight * Yr.
1 80 80/1.08 = 74.07 0.0741 ∗ 1 = 0.0741
2 80 80/1.082 = 68.59 0.0686 ∗ 2 = 0.1372
3 80 80/1.083 = 63.51 0.0635 ∗ 3 = 0.1905
4 80 80/1.084 = 58.80 0.0588 ∗ 4 = 0.2352
5 1080 1080/1.085 = 735.03 0.735 ∗ 5 = 3.6750
Sum $1000 4.312 years

14. Which one of the following par value 12% coupon bonds experiences a price
change of $23 when the market yield changes by 50 basis points?
A) The bond with a duration of 6 years.
B) The bond with a duration of 5 years.
C) The bond with a duration of 2.7 years.
D) The bond with a duration of 5.15 years.
E) None of the above.

Solution. D:
∆P/P = −D × [∆(1 + y)/(1 + y)]
−.023 = −D × [.005/1.12]; D = 5.15.

15. You have an obligation to pay $1, 488 in four years and 2 months. In which
bond would you invest your $1, 000 to accumulate this amount, with relative certainty,
even if the yield on the bond declines to 9.5% immediately after you purchase the
bond?
A) a 6-year; 10% coupon par value bond
B) a 5-year; 10% coupon par value bond
C) a 5-year; zero-coupon bond
D) a 4-year; 10% coupon par value bond
E) none of the above

Solution. B
When duration = horizon date, one is immunized, or protected, against one
interest rate change. The zero has D = 5. Since the other bonds have the same
coupon and yield, solve for the closest value of T that gives D = 4.2 years.
Because the duration of a corporate bond is

1+y (1 + y) + T (c − y)
− ,
y c[(1 + y)T − 1] + y

27
we solve an equation

1.1 (1.1 + T (.1 − .1)


4.2 = −
.1 .1[1.1T − 1] + .1
or [ln(1.10)] = ln(1.6176); T = 5.05 years, so choose the 5-year 10% coupon bond.

16. A bond has a maturity of 2 years. It pays semiannual coupons at annual rate
of 8% and has annual yield of 7%. The par value of the bond is $1,000.
a. What are the convexity and the duration of the bond?
b. Find the actual price of the bond assuming that its yield to maturity
immediately increases from 7% to 7.5% (with maturity still 2 years)
c. What price would be predicted by the duration rule (the formula for the price
change in terms of duration)? What is percentage error of that rule?
d. What price would be predicted by the duration-convexity rule (the formula for
the price change in terms of duration and convexity)? What is percentage error of
that rule?

Solution:
a.
( ) ( )
1+y 1 + y + n(c − y) 1.035 (1.035 + 4(0.04 − 0.035)
D=τ − = 0.5 − ,
y c[(1 + y)n − 1] + y 0.035 0.04(1.0354 − 1) + 0.035
= 1.8887 years
Before the yield change, the price of the bond is P = $1018.37

∑n [ ]
(τ )2 CFi
Convexity = (i + i ) = 4.322 years2 ,
2
P × (1 + y)2 i=1 (1 + y)i
b.
P = $1009.12846
c.
∆P
= −D × ∆y a /(1 + y) = −0.009124,
P
→ Papprox = 1, 009.078
P − Papprox
Error = = 0.0049%
P
d.
∆P 1
= −D⋆ × ∆y a + [Convexity × (∆y a )2 ] = −0.009070,
P 2

28
→ Papprox = $1009.133, Error = −0.00054%

Topic VI
1. Buyers of put options anticipate the value of the underlying asset will
and sellers of call options anticipate the value of the underlying asset will
A) increase; increase
B) decrease; increase
C) increase; decrease
D) decrease; decrease
E) cannot tell without further information

Solution. D: The buyer of the put option hopes the price will fall in order to
exercise the option and sell the stock at a price higher than the market price. Likewise,
the seller of the call option hopes the price will decrease so the option will expire
worthless.

2. The price of stock A is 50$ and its next dividend payment is in 9 months. The
prices of European call and European put options written on stock A are 5$ and 4$,
respectively. The options has 6 months to expiration and the strike price of 50$.
a. At what prices of the stock in 6 months a long investor in the covered call
strategy will have a positive profit?
b. Assuming that the market is efficient, how an investor can create the cash flow
the same as from the covered call strategy by trading the put and T-bills?

Solution. a. The profit from the strategy is equal to its cash flow in six months
minus its cost today. The cost today is 45$ and the cash flow in 6 months is
ST −(ST −50)+ ). It follows that the profit will be positive when ST −(ST −50)+ )−45 >
0. If ST < 50, then the profit is ST − 45 and is positive if ST > 45. If ST ≥ 50 then
the profit is 5$. It follows that the profit will be positive if ST > 45$.

b. Consider the put-call parity:

p + S − pv(D) = c + pv(X)

we can rewrite it as
S − pv(D) − c = −p + pv(X)

29
The present value of the cash flow from the covered call strategy is shown on the
left side of the last equation. It follows that one can replicate the case flow from the
covered call by shorting the put option and buying T-bills for the amount equal to
the present value of 50$.

3. HighFlyer Stock currently sells for $48. A one-year call option with strike price
of $55 sells for $9, and the risk free interest rate is 6%. What is the price of a one-year
put with strike price of $55?
A) $9.00
B) $12.89
C) $16.00
D) $18.72
E) $15.60

Solution. B: We find from the put–call parity P = 9 − 48 + 55/(1.06); P = 12.89

4. The prices of European call and put options on a non-dividend-paying stock


with 12 months to maturity, a strike price of $120, and an expiration date in 12
months are $20 and $5, respectively. The current stock price is $130. What is the
implied risk-free rate?
Solution: From put-call parity

20 + 120/(1 + rf ) = 5 + 130

Solving this
1 + rf = 120/115
so that rf = 4.35%.

5. Use the two-state put option value in this problem. S0 = $100; X = $120; the
two possibilities for ST are $150 and $80. The range of P across the two states is
; the delta is
A) $0 and $40; -4/7
B) $0 and $50; +4/7
C) $0 and $40; +4/7
D) $0 and $50; -4/7
E) $20 and $40; +1/2

Solution. A: When ST = $150; P = $0; When ST = $80 : P = $40. Thus,


($0 − $40)/($150 − $80) = −4/7.

30
6. A portfolio consists of 400 shares of stock and 200 calls on that stock. If the
delta for the call is 0.6, what would be the dollar change in the value of the portfolio
in response to a one dollar decline in the stock price?
A) +$700
B) +$500
C) -$580
D) -$520
E) none of the above

Solution. D: −$400 + [−$200(0.6)] = −$520.

7. A stock price is currently trading at 40. Paul Tripp, CFA wants to value
one-year index option using the single-period binomial model. The stock will either
increase in value by 20% with probability of 40% or fall in value by 30% with
probability of 60%. The annual risk-free interest rate is 2%. No dividends are paid
on any of the underlying securities in the index.
a. Calculate the value of a European call option on the index with an exercise
price of 40$
b. Combine the call options from question a with T-bills and the underlying stock
in the way that the payoff of the resulting portfolio becomes the same as a payoff of a
long straddle with the call option from question a. Find the position in each security
in this portfolio.

Solution. a.
Cu (1 − d + rf ) + Cd (u − 1 − rf )
c= = $5.02,
(u − d)(1 + rf )
where Cu = 40 × 1.2 − 40 = 8, Cd = 0, u = 1.2, d = 0.7.

b. A long straddle includes one call option and one put option both written on
the same stock. It follows that we have to emulate a put option since we cannot trade
it. Consider the put-call parity:

p + S = c + pv(X)

and rewrite it as
p = −S + c + pv(X).
It follows that at any time the cash flow from a put has to be equal to the cash flow
from a short position in the underlying stock plus long position in the call and a long

31
position in T-bills in the amount of pv(X) = $39.22. We conclude that we can create
a long straddle by buying two call options, shorting stock and lending $39.22.
An alternative approach to solving this problem is to create a put option
synthetically by trading the underlying stock and T-bills. This approach is acceptable,
but requires us to assume that the stock follows the binomial model.

8. Consider stock X that follows a single-step binomial tree. The current price of
stock X is 100$ and it can increase to 108$ with probability p = 55% or decrease to
92$ with probability 1 − p = 45% over next 6 months. The effective annual interest
rate is 3%. Suppose that investor Michael writes the call option on stock X with
expiration in 6 months and strike price of 98$.
a. Design a portfolio with stock X and T-bills that Michael has to buy to hedge his
risk (that is, to remove the aggregate risk from the short position in the call and a
long position in the portfolio).
b. Assume that the markets are efficient. What is the expected rate return from
stock X and the call option over 6 months?

Solution:
a. Michael will hedge his risk if he buys a synthetic option. This portfolio should
have ∆ stock shares and Z dollars in T-bills, where
Cu − Cd 10
∆= = = .625
Su − Sd 16
Cd u − Cu d −10 × 0.92
Z= = √ = −56.66$
(u − d)(1 + rf ) T
(1.08 − 0.92) 1 + 0.03
and we took into account that Cu = 10, Cd = 0.

b. In the efficient market the price of the call option is equal to the price of the
synthetic option:

C = ∆ × S0 + Z = 0.625 ∗ 100 − 56.66 = 5.84$.

The expected rate of return on stock X is

E(rS ) = prSu +(1−p)rSd = p(u−1)+(1−p)(d−1) = 0.55×0.08+0.45×(−0.08) = 0.8%

The expected rate of return of the call is:


Cu − C Cd − C
E(rS ) = prCu + (1 − p)rCd = p + (1 − p)
C C
10 − 5.84 0 − 5.84
= 0.55 × + 0.45 × = −5.82%
5.84 5.84

32

You might also like

pFad - Phonifier reborn

Pfad - The Proxy pFad of © 2024 Garber Painting. All rights reserved.

Note: This service is not intended for secure transactions such as banking, social media, email, or purchasing. Use at your own risk. We assume no liability whatsoever for broken pages.


Alternative Proxies:

Alternative Proxy

pFad Proxy

pFad v3 Proxy

pFad v4 Proxy